A large number of drivers routinely (Assumption CR)

This topic has expert replies
User avatar
Legendary Member
Posts: 1560
Joined: Tue Nov 17, 2009 2:38 am
Thanked: 137 times
Followed by:5 members
A large number of drivers routinely violate highway speed limits. Since driving at speeds that exceed posted limits is a significant factor in most accidents, installing devices in all cars that prevent those cars from traveling faster than the speed limit would prevent most accidents.
Which one of the following is an assumption on which the argument depends?
(A) A person need not be a trained mechanic to install the device properly.
(B) Most accidents are caused by inexperienced drivers.
(C) A driver seldom needs to exceed the speed limit to avoid an accident when none of the other drivers involved are violating the speed limit.
(D) Most drivers who exceed the speed limit do so unintentionally.
(E) Even if the fines for speed-limit violations were increased, the number of such violations would still not be reduced.
Many of the great achievements of the world were accomplished by tired and discouraged men who kept on working

User avatar
Legendary Member
Posts: 758
Joined: Sat Aug 29, 2009 9:32 pm
Location: Bangalore,India
Thanked: 67 times
Followed by:2 members

by sumanr84 » Mon May 03, 2010 10:41 pm
thephoenix wrote:A large number of drivers routinely violate highway speed limits. Since driving at speeds that exceed posted limits is a significant factor in most accidents, installing devices in all cars that prevent those cars from traveling faster than the speed limit would prevent most accidents.
Which one of the following is an assumption on which the argument depends?
(A) A person need not be a trained mechanic to install the device properly.
(B) Most accidents are caused by inexperienced drivers.
(C) A driver seldom needs to exceed the speed limit to avoid an accident when none of the other drivers involved are violating the speed limit.
(D) Most drivers who exceed the speed limit do so unintentionally.
(E) Even if the fines for speed-limit violations were increased, the number of such violations would still not be reduced.
I think its b/w D and E.
My pick is E. Author assumes that the fine cannot control speed limit violations.So, he promotes installing devices.
D uses MOST but not ALL for the assumption to hold good for all drivers.
I am on a break !!

User avatar
Legendary Member
Posts: 758
Joined: Sat Aug 29, 2009 9:32 pm
Location: Bangalore,India
Thanked: 67 times
Followed by:2 members

by sumanr84 » Mon May 03, 2010 11:13 pm
I think I proceeded in the wrong direction. Focus should be on the conclusion "how to prevent accident ". Author says that accidents are caused by speed limit violation. So, his assumption is that if we don't exceed the speed limit there will be no accident. Only, C looks near to that assumption.

someone pls put more light on this ?
I am on a break !!

GMAT Instructor
Posts: 1302
Joined: Mon Oct 19, 2009 2:13 pm
Location: Toronto
Thanked: 539 times
Followed by:164 members
GMAT Score:800

by Testluv » Mon May 03, 2010 11:23 pm
sumanr84 wrote:I think I proceeded in the wrong direction. Focus should be on the conclusion "how to prevent accident ". Author says that accidents are caused by speed limit violation. So, his assumption is that if we don't exceed the speed limit there will be no accident. Only, C looks near to that assumption.

someone pls put more light on this ?
Yep, good reasonoing. Because the question asks for an assumption on which the argument depends, we can use the Kaplan denial test to check choice C. Choice C reads:
C) A driver seldom needs to exceed the speed limit to avoid an accident when none of the other drivers involved are violating the speed limit.


whose denial would be:

"A driver often needs to exceed the speed limit to avoid an accident when none of the other drivers involved are violating the speed limit"

in which case installation of these devices may actually cause rather than prevent accidents as the argument asserts.

Because in the absence of choice C the argument falls apart, choice C is an assumption on which the argument depends. As an aside, all four wrong answers are outside the scope of the argument.
Kaplan Teacher in Toronto

Junior | Next Rank: 30 Posts
Posts: 10
Joined: Mon Jun 21, 2010 8:03 am

by Operation780 » Thu Aug 05, 2010 7:19 am
In didnot understand why (E) is wrong.
(E) Even if the fines for speed-limit violations were increased, the number of such violations would still not be reduced.

Fact is "driving at speeds that exceed posted limits is a significant factor in most accidents. i.e. exceed posted limits -> most accidents"
Conclusion is "installing devices in all cars that prevent those cars from traveling faster than the speed limit would prevent most accidents. i.e. install devices -> no exceed of posted limits ->prevent most accidents. hence install devices -> prevent most accidents.

If we negate (E), it would read as "if fines for speed-limit violations were increased, the number of such violations would be reduced i.e. increase fine -> no exceed of posted limits ->prevent most accidents".
Hence negating E would weaken the conclusion.

PLease explain

User avatar
Legendary Member
Posts: 1261
Joined: Sun Sep 14, 2008 3:46 am
Thanked: 27 times
GMAT Score:570

by reply2spg » Thu Aug 05, 2010 8:32 am
Can you please elaborate C in detail?
Testluv wrote:
sumanr84 wrote:I think I proceeded in the wrong direction. Focus should be on the conclusion "how to prevent accident ". Author says that accidents are caused by speed limit violation. So, his assumption is that if we don't exceed the speed limit there will be no accident. Only, C looks near to that assumption.

someone pls put more light on this ?
Yep, good reasonoing. Because the question asks for an assumption on which the argument depends, we can use the Kaplan denial test to check choice C. Choice C reads:
C) A driver seldom needs to exceed the speed limit to avoid an accident when none of the other drivers involved are violating the speed limit.


whose denial would be:

"A driver often needs to exceed the speed limit to avoid an accident when none of the other drivers involved are violating the speed limit"

in which case installation of these devices may actually cause rather than prevent accidents as the argument asserts.

Because in the absence of choice C the argument falls apart, choice C is an assumption on which the argument depends. As an aside, all four wrong answers are outside the scope of the argument.
Sudhanshu
(have lot of things to learn from all of you)

GMAT Instructor
Posts: 1302
Joined: Mon Oct 19, 2009 2:13 pm
Location: Toronto
Thanked: 539 times
Followed by:164 members
GMAT Score:800

by Testluv » Thu Aug 26, 2010 9:07 pm
Can you please elaborate C in detail?


We can find an assumption by negating a valid objection to the argument. (Objection hurts an argument. So, negating an objection helps an argument. An assumption is unstated evidence--and evidence helps an argument).

The author is arguing that by disallowing drivers from exceeding the speed limit, we will prevent most accidents.

How can we object?

Well, what if drivers sometimes need to exceed the speed limit in order to avoid an accident? In that case, the author's argument is not sound.

So, the author is assuming that drivers DON'T need to exceed the speed limit in order to avoid an accident. This is what choice C says.

Choice D is irrelevant/outside the scope: we don't care whether drivers mean to exceed speed limit or not. We care about whether exceeding speed limit is the cause of most accidents.

Choice E is irrelevant/outside the scope: we don't care about fines.
Kaplan Teacher in Toronto

Master | Next Rank: 500 Posts
Posts: 364
Joined: Tue Apr 20, 2010 5:13 am
Thanked: 31 times
Followed by:3 members

by FightWithGMAT » Thu Aug 26, 2010 11:30 pm
thephoenix wrote:A large number of drivers routinely violate highway speed limits. Since driving at speeds that exceed posted limits is a significant factor in most accidents, installing devices in all cars that prevent those cars from traveling faster than the speed limit would prevent most accidents.
Which one of the following is an assumption on which the argument depends?
(A) A person need not be a trained mechanic to install the device properly.
(B) Most accidents are caused by inexperienced drivers.
(C) A driver seldom needs to exceed the speed limit to avoid an accident when none of the other drivers involved are violating the speed limit.
(D) Most drivers who exceed the speed limit do so unintentionally.
(E) Even if the fines for speed-limit violations were increased, the number of such violations would still not be reduced.
C simply shows that the reverse of cause-effect is not the case.

Argument says that Over-speeding causes accidents.

C says that to avoid accidents, people rarely over speed.

Negate this:

To avoid accidents, people generally over speed.